悠闲数学娱乐论坛(第2版)'s Archiver

青青子衿 发表于 2018-12-2 13:45

一道反正切函数的级数问题

[i=s] 本帖最后由 青青子衿 于 2018-12-2 13:46 编辑 [/i]

\[\color{black}{\sum\limits_{n=1}^{\infty}\arctan\frac{1}{n^2+1}=\arctan\left(\frac{\tan\left(\sqrt{\frac{\sqrt{2}-1}{2}}\pi\right)}{\tanh\left(\sqrt{\frac{\sqrt{2}+1}{2}}\pi\right)}\right)-\frac{\pi}{8}}\]

色k 发表于 2018-12-2 14:49

之前好像有帖子讨论过(或是类似的)……

isee 发表于 2018-12-2 15:06

[b]回复 [url=http://kuing.orzweb.net/redirect.php?goto=findpost&pid=28997&ptid=5766]2#[/url] [i]色k[/i] [/b]


    [url]http://kuing.orzweb.net/viewthread.php?tid=4922[/url]

战巡 发表于 2018-12-4 10:00

[i=s] 本帖最后由 战巡 于 2018-12-6 00:13 编辑 [/i]

[b]回复 [url=http://kuing.orzweb.net/redirect.php?goto=findpost&pid=28996&ptid=5766]1#[/url] [i]青青子衿[/i] [/b]

\[\arctan(\frac{1}{n^2+1})=\frac{i}{2}\ln(\frac{i+\frac{1}{n^2+1}}{i-\frac{1}{n^2+1}})=\frac{i}{2}\ln(1-\frac{2i}{n^2+(i+1)})\]
\[\sum_{n=1}^\infty\arctan(\frac{1}{n^2+1})=\frac{i}{2}\ln[\prod_{n=1}^\infty(1-\frac{2i}{n^2+(i+1)})]\]
然后,由于
\[1-\frac{2i}{n^2+(i+1)}=\frac{1-\frac{(2i-1-i)\pi^2}{\pi^2n^2}}{1+\frac{(i+1)\pi^2}{\pi^2n^2}}\]
上面变成
\[=\frac{i}{2}\ln[\prod_{n=1}^\infty\frac{1-\frac{(2i-1-i)\pi^2}{\pi^2n^2}}{1+\frac{(i+1)\pi^2}{\pi^2n^2}}]\]
另一方面知道
\[\sin(x)=x\prod_{n=1}^\infty(1-\frac{x^2}{\pi^2n^2}),\sinh(x)=x\prod_{n=1}^\infty(1+\frac{x^2}{\pi^2n^2})\]
有上面
\[=\frac{i}{2}\ln[\frac{\sqrt{i+1}}{\sqrt{i-1}}\frac{\sin(\sqrt{i-1}\pi)}{\sinh(\sqrt{i+1}\pi)}]\]
剩下懒的搞了...连接里面引用的那本书上的方法应该可以继续化简

战巡 发表于 2018-12-6 01:00

[b]回复 [url=http://kuing.orzweb.net/redirect.php?goto=findpost&pid=28996&ptid=5766]1#[/url] [i]青青子衿[/i] [/b]


也罢,给你做完吧

这里不难证明
\[\sqrt{i-1}=a+bi, \sqrt{i+1}=b+ai\]
其中
\[a=\sqrt{\frac{\sqrt{2}-1}{2}},b=\sqrt{\frac{\sqrt{2}+1}{2}}\]

\[\frac{\sin(\sqrt{i-1}\pi)}{\sinh(\sqrt{i+1}\pi)}=\frac{\sin(a\pi)\cos(bi\pi)+\sin(bi\pi)\cos(a\pi)}{\sinh(b\pi)\cosh(ai\pi)+\cosh(b\pi)\sinh(ai\pi)}\]
\[=\frac{\sin(a\pi)\cosh(b\pi)+i\sinh(b\pi)\cos(a\pi)}{\sinh(b\pi)\cos(a\pi)+i\cosh(b\pi)\sin(a\pi)}=\frac{\tan(a\pi)+i\tanh(b\pi)}{\tanh(b\pi)+i\tan(a\pi)}\]
\[=\frac{i+\frac{\tan(a\pi)}{\tanh(b\pi)}}{i-\frac{\tan(a\pi)}{\tanh(b\pi)}}·\frac{i-1}{i+1}\]

那么原式
\[=\frac{i}{2}\ln[\frac{\sqrt{i+1}}{\sqrt{i-1}}·\frac{i+\frac{\tan(a\pi)}{\tanh(b\pi)}}{i-\frac{\tan(a\pi)}{\tanh(b\pi)}}·\frac{i-1}{i+1}]\]
\[=\frac{i}{2}\ln[\frac{i+\frac{\tan(a\pi)}{\tanh(b\pi)}}{i-\frac{\tan(a\pi)}{\tanh(b\pi)}}]-\frac{i}{2}\ln[\frac{\sqrt{i+1}}{\sqrt{i-1}}]=\arctan(\frac{\tan(a\pi)}{\tanh(b\pi)})-\frac{\pi}{8}\]

青青子衿 发表于 2018-12-6 18:51

[b]回复 [url=http://kuing.orzweb.net/redirect.php?goto=findpost&pid=29034&ptid=5766]5#[/url] [i]战巡[/i] [/b]
谢谢站版主{:handshake:}

页: [1]

Powered by Discuz! Archiver 7.2  © 2001-2009 Comsenz Inc.